Network Analysis/Loop Rule Problem

  • Thread starter hagobarcos
  • Start date
  • Tags
    Network
In summary: Eq 2 For the top left loop: i1 + i6 = i4And for the top right loop:i1 + i5 = i3And for the bottom left loop:i1 + i10 = i7And for the bottom right loop:i1 + i9 = i8And for the center loop:i1 + i4 = 0
  • #1
hagobarcos
34
0

Homework Statement



This is out of a book on Excel, where we are solving simultaneous equations. However, how do we find the equations for this network in the first place? (image at bottom)

Homework Equations



∑ I= 0

∑ V =0

The Attempt at a Solution



i1 + i2 + i3 = 2

i6 + i7 +i10 = 3
 

Attachments

  • IMG_0479.jpg
    IMG_0479.jpg
    35.3 KB · Views: 422
Last edited:
Physics news on Phys.org
  • #2
hagobarcos said:

Homework Statement



This is out of a book on Excel, where we are solving simultaneous equations. However, how do we find the equations for this network in the first place? (image at bottom)




Homework Equations



∑ I= 0

∑ V =0


The Attempt at a Solution



i1 + i2 + i3 = 2

i6 + i7 +i10 = 3


LOL, no attempt at solution no help...

First start making equations at various junctions using Kirchoff's Junction Rule.
No, I will not start making them myself... -_-
 
  • #3
oops! sorry, did not mean to add all that. Will edit in a second.

ok, so for top point junction, i1 + i2 +i3 = 2

and i6 + i10 + i 7 = 3 for bottom left junction.

Am stuck on the rest.
 
  • #4
hagobarcos said:
oops! sorry, did not mean to add all that. Will edit in a second.

ok, so for top point junction, i1 + i2 +i3 = 2

and i6 + i10 + i 7 = 3 for bottom left junction.

Am stuck on the rest.

You need 4 more equations of KJL. Come on, its current inwards = current outwards. Make them.

Other is i1+i6=i4...

Make 3 more...
 
  • Like
Likes 1 person
  • #5
Hmmm,

so for bottom right junction:

i10 + i9 = i8

And then top right junction:

i5 = i3 + i9

And for the Center:

Wait, what to do for center? Is the current flowing in all dribbling out into the ground?

Guessing, by conservation of energy:

i4 + i5 + i8 + i7 = 5
 
  • #6
hagobarcos said:
Hmmm,

so for bottom right junction:

i10 + i9 = i8

And then top right junction:

i5 = i3 + i9

And for the Center:

Wait, what to do for center? Is the current flowing in all dribbling out into the ground?

Guessing, by conservation of energy:

i4 + i5 + i8 + i7 = 5

Hmmm, no.

That thing is connected to the earth. So at center, sum of all currents equals zero.

Now you have 6 equations but 10 variables. Applying Kirchoff's Voltage rule for 4 loops will give you 4 more equations and consequently you'll have 10 variables and 10 equations...
 
  • Like
Likes 1 person
  • #7
Uh oh. Okay.

SO, then for the center, i4 + i5 +i8 +i7 = 0

And then for the loops, assuming current is flowing clockwise:

Top left loop:

(+10)i1 -(40)i2 + (10)i4 = 0

Top right loop:

(+40)i2 - (5)i3 - (20)i5 = 0

Bottom right loop:

(-10)i9 -(10)i8 +(20)i5 = 0

Center loop:

(+10)i8 + (5)i10 -(50)i7 = 0

Bottom left Loop:

(-10)i4 + (50)i7 - (2)i6 = 0
 
  • #8
sankalpmittal said:
Hmmm, no.

That thing is connected to the earth. So at center, sum of all currents equals zero.

That's not right. It must be assumed that the currents flowing into the ground node will sum to the external currents entering at the nodes where currents are shown entering the circuit. Otherwise there will be a net buildup of charge on the circuit coming from the external sources, and that can't be right.

If you like, imagine 2 A and 3 A current sources connected between ground and those nodes where the 2 A and 3A currents are shown "coming in from nowhere".
 
  • #9
Oh man this is hard :D

Yes I thought as much.

Thus: Ground node equation :

i4 + i5 + i8 + i7 = 5

However, this one is not mentioned in the answers. So it may be redundant.

AND, question, is it possible for the current to flow backwards into the source?
 
  • #10
hagobarcos said:
Oh man this is hard :D

Yes I thought as much.

Thus: Ground node equation :

i4 + i5 + i8 + i7 = 5
Yup! But don't forget i2.

However, this one is not mentioned in the answers. So it may be redundant.

AND, question, is it possible for the current to flow backwards into the source?

You should assume that the external currents shown are correct in their directions. The other branch currents are unknowns so their actual directions are not necessarily as assumed in the diagram. {I presume that you are meant to solve for the individual branch currents}.

You might find it convenient to write Node Equations (see: Nodal Analysis) to first determine the node potentials with respect to the ground node. There will be five equations in five unknowns. Then use the node potentials to determine the individual branch currents (Ohm's Law).
 
  • #11
Yes.

Revised Ground Node eq: i2+ i5 + i8 + i7 + i4 = 5 amps

Moving on to system of equations:

For the nodes/corners of the pentagon:

i1 + i6 = i4

So

i1 + i6 - i4 = 0 ----Eq 1

2= i1 + i2 + i3

So

i1 + i2 + i3 = 2 ----Eq 2

i3 = i5 + i9

So

i3 - i5 - i9 = 0 ----Eq 3

i9 + i10 = i8

so

i9 + i10 - i8 = 0 ---- Eq 4

3 = i10 + i7 + i6

so

i10 + i7 + i6 = 3 ---- Eq 5

For the loops, using V = IR, and ∑ V = 0 :

(-2)(i6) +(-10)(i4) + (50)(i7) = 0 ----Eq 6

(10)(i1) + (-40)(i2) + (10)(i4) = 0 ----Eq 7

(-5)(i3) + (-20)(i5) + (40)(i2) = 0 ----Eq 8

(20)(i5) + (-10)(i9) + (-10)(i8) = 0 ---- Eq 9

(10)(i8) + (5)(i10) + (-50)(i7) = 0 ----Eq 10


Only problem now is, a few of my equations, namely Eq 6 - Eq 10 are backwards. Attached below are the answers that I am trying to match, although I am not sure how to modify mine without messing it up.
 

Attachments

  • IMG_8930.jpg
    IMG_8930.jpg
    46 KB · Views: 391
  • #12
I've looked (quickly) over your loop equations, and they appear to be fine. I'm not sure what you mean by some of them being "backwards". Keep in mind that you may multiply an entire equation through by -1 and it will still be the same equation...
 
  • Like
Likes 1 person
  • #13
Bah. Right on. Yay that means it is all ready for matrix solving.
Thank you guys!
 
  • #14
Thanks for the catch Gneill ! :redface:
 

Related to Network Analysis/Loop Rule Problem

What is network analysis/loop rule problem?

Network analysis, also known as circuit analysis, is the process of studying and solving electrical circuits using mathematical techniques. The loop rule problem is a specific type of network analysis problem that involves analyzing the voltage and current in a closed loop or circuit.

What is the loop rule in network analysis?

The loop rule, also known as Kirchhoff's voltage law, states that the sum of the voltage drops around a closed loop in a circuit must equal the sum of the voltage sources in that loop. This allows for the calculation of unknown voltages in a circuit.

How do you apply the loop rule to solve a network analysis problem?

To apply the loop rule, you must first identify all the voltage sources and voltage drops in the circuit. Then, assign a direction to the current flow in each loop. Next, write out the loop rule equation by summing the voltage drops and setting it equal to the sum of the voltage sources. Finally, solve the equation to find the unknown voltage.

What is the difference between the loop rule and the node rule?

The loop rule deals with the voltage drops and sources around a closed loop in a circuit, while the node rule, or Kirchhoff's current law, deals with the current entering and exiting a node or junction in a circuit. Both rules are essential in solving network analysis problems.

Why is network analysis important in science?

Network analysis is crucial in science because it allows us to understand and predict the behavior of complex electrical circuits. It is used in various fields such as engineering, physics, and computer science to design and troubleshoot circuits and systems. It also helps in identifying potential problems and optimizing circuit performance.

Similar threads

  • Introductory Physics Homework Help
Replies
1
Views
695
  • Introductory Physics Homework Help
Replies
9
Views
1K
  • Introductory Physics Homework Help
Replies
1
Views
802
  • Introductory Physics Homework Help
Replies
5
Views
3K
  • Introductory Physics Homework Help
Replies
9
Views
2K
  • Introductory Physics Homework Help
Replies
6
Views
868
  • Introductory Physics Homework Help
Replies
3
Views
1K
  • Introductory Physics Homework Help
Replies
4
Views
1K
  • Introductory Physics Homework Help
Replies
31
Views
2K
  • Introductory Physics Homework Help
Replies
11
Views
2K
Back
Top